图论(第五版)reinhard-diestel著 于青林译 第一章练习七解答备忘

考虑没有孤立顶点的无向简单图 G G G(通常假设 G G G连通,但在这里条件被放宽),设 G G G的任意一个最小圈为 C C C

C C C的长度为 2 r + 1 2r+1 2r+1 ,在 C C C上任取一点 s 0 s_{0} s0 ,剩下 2 r 2r 2r 个点拆成连续相对的两组,每组 r r r个点(从 s 0 s_{0} s0 任意一侧出发沿 C C C绕行遇满 r r r个顶点后把这 r r r个顶点作为一组,其余 r r r个顶点作为另外一组),两组的点和 s 0 s_{0} s0 分别构成 C C C上的两条路径 P 1 , P 2 P1,P2 P1,P2,长度均为 r r r

s 0 s_{0} s0 所在的 G G G的连通分支构成的子图是 G ′ ( V ′ , E ′ ) G'(V', E') G(V,E)

注意对于一个最小圈,如果用圈上两个不同的点把圈划分成两条路径,则长度较短的那条路径必然是最短路径,

由此不难推得 P 1 , P 2 P1,P2 P1,P2均为 G ′ G' G的最短路径

现令 d ′ = m a x v ∈ V ′ d G ′ ( s 0 , v ) d'=max_{v\in V'}{d_{G'}(s_{0}, v)} d=maxvVdG(s0,v) , d [ s 0 , i ] d[s_{0},i] d[s0,i] 表示 G ′ G' G中所有和 s 0 s_{0} s0 的最短距离为i的顶点的集合( 0 < = i < = d ′ 0<=i<=d' 0<=i<=d )

d [ s 0 , i ] ≠ ⊘ d[s_{0}, i]\ne \oslash d[s0,i]= 0 < = i < = d ′ 0<=i<=d' 0<=i<=d d [ s 0 , i ] ∩ d [ s 0 , j ] = ⊘ d[s_{0},i]\cap d[s_{0},j] = \oslash d[s0,i]d[s0,j]= 0 < = i < j < = d ′ 0<=i<j<=d' 0<=i<j<=d V ′ = ⋃ i = 0 d ′ d [ s 0 , i ] V' = \bigcup_{i = 0}^{d'}{d[s_{0}, i]} V=i=0dd[s0,i]

注意 C ⊆ G ′ C\subseteq G' CG ,由于P1,P2均为 G ′ G' G的最短路径,所以 r < = d ′ r<=d' r<=d

现在我们来证明命题 P ( i ) P(i) P(i)( 0 < = i < r 0<=i<r 0<=i<r)

对于 d [ s 0 , i ] d[s_{0},i] d[s0,i] 中的任意顶点 v v v,与 v v v关联的所有边中只有一条和 d [ s 0 , i − 1 ] d[s_{0},i-1] d[s0,i1] 中的顶点相连(如果存在,即 i i i不等于0),其余(如果有)全部和 d [ s 0 , i + 1 ] d[s_{0}, i+1] d[s0,i+1] 中的顶点相连,并且对任意 s 1 s 2 ∈ d [ s 0 , i ] s_1 s_2 \in d[s_{0}, i] s1s2d[s0,i] ( s 1 ≠ s 2 i ≠ 0 s_1 \ne s_2 i\ne 0 s1=s2i=0 ) 有 ( N G ′ ( s 1 ) − ( s 1 ′ ) ) ∩ ( N G ′ ( s 2 ) − ( s 2 ′ ) N_{G'}(s_1)-(s'_1)) \cap (N_{G'}(s_2)-(s'_2) NG(s1)(s1))(NG(s2)(s2)) = ⊘ = \oslash = 其中 s 1 ′ s'_1 s1 是与 s 1 s_1 s1 d [ s 0 , i − 1 ] d[s_0, i-1] d[s0,i1] 中相邻的顶点, s 2 ′ s'_2 s2 同理

事实上,当 i = 0 i=0 i=0时, d [ s 0 , 0 ] = s 0 d[s_0, 0] = s_0 d[s0,0]=s0 ,而与 s 0 s_0 s0 关联的任意一个顶点到 s 0 s_0 s0 的最短距离均为1,因而必然在 d [ s 0 , 1 ] d[s_0,1] d[s0,1]

现设 i = k i=k i=k时命题 P ( i ) P(i) P(i)成立,下面来证 i = k + 1 i=k+1 i=k+1时命题 P ( i ) P(i) P(i)成立

事实上, d [ s 0 , k + 1 ] d[s_0, k+1] d[s0,k+1] 中的顶点 v v v不可能和 d [ s 0 , i 1 ] d[s_0,i_1] d[s0,i1] 0 < = i 1 < k 0<=i_1<k 0<=i1<k d [ s 0 , i 2 ] d[s_0, i_2] d[s0,i2] k + 2 < i 2 < = d ′ k+2<i_2 <=d' k+2<i2<=d (如果 i 1 , i 2 i1,i2 i1,i2存在)中的顶点邻接,如果 v v v d [ s 0 , i 1 ] d[s_0,i_1] d[s0,i1] 中的顶点邻接,则存在一条长为 i 1 + 1 i_1+1 i1+1 的从 s 0 s_0 s0 v v v的路径,从而 v ∈ d [ s 0 , i 1 + 1 ] v\in d[s_0, i_1+1] vd[s0,i1+1] ,但 i 1 + 1 < = k i_1+1<=k i1+1<=k 矛盾,同理如果 v v v d [ s 0 , i 2 ] d[s_0, i_2] d[s0,i2] 中的顶点 w w w邻接,则存在一条从 s 0 到 w s_0 到w s0w的路径,该路径的长为 k + 2 k+2 k+2,于是 w ∈ d [ s 0 , k + 2 ] w \in d[s_0,k+2] wd[s0,k+2] ,但 k + 2 < i 2 k+2<i_2 k+2<i2 矛盾

另外,由于 s 0 s_0 s0 v v v的最短路径长度为 k + 1 k+1 k+1,所以 s 0 s_0 s0 到该最短路径上 v v v的前驱顶点 w w w的最短路径长为 k k k,故 w ∈ d [ s 0 , k ] w \in d[s_0,k] wd[s0,k]

但由归纳假设, d [ s 0 , k ] d[s_0,k] d[s0,k] 上不同的顶点除去 d [ s 0 , k − 1 ] d[s_0,k-1] d[s0,k1] (如果存在)中的顶点的邻点集互不相交,因此 v v v不可能和

d [ s 0 , k ] d[s_0,k] d[s0,k] 中两个或两个以上的顶点邻接,因此 v v v只和 d [ s 0 , k ] d[s_0, k] d[s0,k] 中唯一一个顶点邻接,于是综上所述,与 v v v关联的剩下所有顶点(如果有)只可能在 d [ s 0 , k + 2 ] d[s_0,k+2] d[s0,k+2] 中,如果存在

s 1 s 2 ∈ d [ s 0 , k + 1 ] s_1s_2\in d[s_{0}, k+1] s1s2d[s0,k+1] ( s 1 ≠ s 2 s_1 \ne s_2 s1=s2 ) 有顶点 w ∈ ( N G ′ ( s 1 ) − ( s 1 ′ ) ) ∩ ( N G ′ ( s 2 ) − ( s 2 ′ ) ) w\in (N_{G'}(s_1)-(s'_1))\cap (N_{G'}(s_2)-(s'_2)) w(NG(s1)(s1))(NG(s2)(s2))其中 s 1 ′ s'_1 s1 是与 s 1 s_1 s1 d [ s 0 , k ] d[s_0, k] d[s0,k] 中相邻的顶点, s 2 ′ s'_2 s2 同理

那么由之前已证明的结论, w ∈ d [ s 0 , k + 2 ] w\in d[s_0, k+2] wd[s0,k+2] ,记 s 1 s_1 s1 s 0 s_0 s0 长为 k + 1 k+1 k+1的路径为P1’, s 2 s_2 s2 s 0 s_0 s0 长为 k + 1 k+1 k+1的路径为P2’,记P1’和P2’上最后一个公共顶点为 t t t,则P1’上 t t t s 1 s_1 s1路径和 s 1 s_1 s1 w w w s 2 s_2 s2 的路径以及P2’上 s 2 s_2 s2 t t t的路径构成了一个圈 C ′ C' C,其长度 L < = l e n g t h ( P 1 ′ ) + l e n g t h ( P 2 ′ ) + 1 + 1 < = 2 ( k + 1 ) + 2 = 2 ( k + 2 ) < = 2 r L<=length(P1')+length(P2') + 1 + 1 <=2(k+1)+2=2(k+2)<=2r L<=length(P1)+length(P2)+1+1<=2(k+1)+2=2(k+2)<=2r

这和 C C C G ′ G' G的最小圈矛盾 这就证明了 i = k + 1 i=k+1 i=k+1时命题 P ( i ) P(i) P(i)成立,由数学归纳法,命题 P ( i ) P(i) P(i) 0 < = i < r 0<=i<r 0<=i<r均成立

现在来导出练习所要求的第一个结论

事实上, s 0 s_0 s0 是我们要考虑的第一个顶点其个数为1, ∣ d [ s 0 , 0 ] ∣ = 1 |d[s_0,0]|=1 d[s0,0]=1

由命题 P ( 0 ) P(0) P(0), s 0 s_0 s0 的所有邻接顶点均位于 d [ s 0 , 1 ] d[s_0,1] d[s0,1] 中,故 ∣ d [ s 0 , 1 ] ∣ > = δ ( G ′ ) > = δ ( G ) |d[s_0,1]|>=\delta(G')>=\delta(G) d[s0,1]>=δ(G)>=δ(G)

2 < = i < = r 2<=i<=r 2<=i<=r,由命题 P ( i − 1 ) , d [ s 0 , i − 1 ] P(i-1), d[s_0,i-1] P(i1),d[s0,i1] 中的每一个顶点的邻点集除一个顶点外,其余均位于 d [ s 0 , i ] d[s_0,i] d[s0,i] 中,且不同顶点的邻点集除位于 d [ s 0 , i − 2 ] d[s_0,i-2] d[s0,i2] 中的一个顶点外其余顶点构成的集合互不相交,故

∣ d [ s 0 , i ] ∣ > = ∑ v ∈ d [ s 0 , i − 1 ] ∣ N G ′ ( v ) − ( v ′ ) ∣ > = ∑ v ∈ d [ s 0 , i − 1 ] δ ( G ′ ) − 1 > = ∣ d [ s 0 , i − 1 ] ∣ ( δ ( G ) − 1 ) |d[s_0, i]|>= \sum_{v\in d[s_0, i-1]}{|N_{G'}(v)-(v')|}>=\sum_{v\in d[s_0, i-1]}{\delta(G')-1} >= |d[s_0,i-1]|(\delta(G)-1) d[s0,i]>=vd[s0,i1]NG(v)(v)>=vd[s0,i1]δ(G)1>=d[s0,i1](δ(G)1)

其中 v ′ v' v v v v d [ s 0 , i − 2 ] d[s_0,i-2] d[s0,i2] 中的唯一邻接顶点

从而 ∣ d [ s 0 , i ] ∣ > = ∣ d [ s 0 , i − 1 ] ∣ ( δ ( G ) − 1 ) > = ∣ d [ s 0 , i − 2 ] ∣ ( δ ( G ) − 1 ) 2 > = − − − > = |d[s_0,i]|>= |d[s_0,i-1]|(\delta(G)-1)>= |d[s_0,i-2]|(\delta(G)-1)^{2} >=--->= d[s0,i]>=d[s0,i1](δ(G)1)>=d[s0,i2](δ(G)1)2>=>=

∣ d [ s 0 , 1 ] ∣ ( δ ( G ) − 1 ) i − 1 |d[s_0,1]|(\delta(G)-1)^{i-1} d[s0,1](δ(G)1)i1

于是 V ( G ) > = V ( G ′ ) = ∣ d [ s 0 , 0 ] ∣ + ∣ d [ s 0 , 1 ] ∣ + ∑ i = 2 r ∣ d [ s 0 , i ] ∣ + ∑ i = r + 1 d ′ ∣ d [ s 0 , i ] ∣ > = ∣ d [ s 0 , 0 ] ∣ + ∣ d [ s 0 , 1 ] ∣ + V(G)>=V(G')= |d[s_0,0]|+|d[s_0,1]|+\sum_{i=2}^{r}|d[s_0,i]|+ \sum_{i = r+1}^{d'}{|d[s_0,i]|}>= |d[s_0,0]|+|d[s_0,1]|+ V(G)>=V(G)=d[s0,0]+d[s0,1]+i=2rd[s0,i]+i=r+1dd[s0,i]>=d[s0,0]+d[s0,1]+ ∑ i = 2 r ∣ d [ s 0 , 1 ] ∣ ( δ ( G ) − 1 ) i − 1 + ∑ i = r + 1 d ′ ∣ d [ s 0 , i ] ∣ \sum_{i=2}^{r}|d[s_0,1]|(\delta(G)-1)^{i-1} + \sum_{i = r+1}^{d'}{|d[s_0,i]|} i=2rd[s0,1](δ(G)1)i1+i=r+1dd[s0,i]

> = ∣ d [ s 0 , 0 ] ∣ + ∑ i = 1 r ∣ d [ s 0 , 1 ] ∣ ( δ ( G ) − 1 ) i − 1 > = 1 + ∑ i = 1 r δ ( G ) ( δ ( G ) − 1 ) i − 1 = 1 + δ ( G ) ∑ i = 1 r ( δ ( G ) − 1 ) i − 1 = 1 + δ ( G ) ∑ i = 0 r − 1 ( δ ( G ) − 1 ) i >= |d[s_0,0]|+\sum_{i=1}^{r}|d[s_0,1]|(\delta(G)-1)^{i-1} >= 1+ \sum_{i=1}^{r}\delta(G)(\delta(G)-1)^{i-1} = 1+ \delta(G)\sum_{i=1}^{r}(\delta(G)-1)^{i-1} = 1+\delta(G)\sum_{i=0}^{r-1}(\delta(G)-1)^{i} >=d[s0,0]+i=1rd[s0,1](δ(G)1)i1>=1+i=1rδ(G)(δ(G)1)i1=1+δ(G)i=1r(δ(G)1)i1=1+δ(G)i=0r1(δ(G)1)i

这样我们就得到了练习要求的第一个结论

至于第二个结论,沿用之前的记号,此时 C C C的长度为2r,在 C C C上任取相邻的两点 s 0 , s 0 ′ s_0,s_0' s0,s0,同样设 s 0 , s 0 ′ s_0,s_0' s0,s0 所在的连通分支为 G ′ ( V ′ , E ′ ) G'(V',E') G(V,E),把从 s 0 s_0 s0 出发从不是 s 0 ′ s_0' s0 所在那一侧沿 C C C行进遇到的 r − 1 r-1 r1个非 s 0 s_0 s0 顶点和 s 0 s_0 s0 构成的路径为P1, C C C上除P1外的其余顶点构成的路径为P2,于是P1,P2均为最短路径.且P1加上 s 0 ′ s_0' s0 和P2加上 s 0 s_0 s0 也是最短路径,这样定义

d [ s 0 , s 0 ′ , i ] d[s_0,s_0',i] d[s0,s0,i] 表示距离 s 0 s_0 s0 s 0 ′ s'_0 s0 最短路径长度中的较小值为 i i i G ′ G' G中的顶点的集合,令 d ′ = m a x v ∈ V ′ ( m i n ( d G ′ ( s 0 , v ) , d G ′ ( s 0 ′ , v ) ) ) d'=max_{v\in V'}(min(d_{G'}(s_0,v), d_{G'}(s'_0, v))) d=maxvV(min(dG(s0,v),dG(s0,v))) 于是 0 < = i < = d ′ 0<=i<=d' 0<=i<=d

显然 d [ s 0 , s 0 ′ , 0 ] = ( s 0 , s 0 ′ ) d[s_0,s_0',0]=(s_0,s'_0) d[s0,s0,0]=(s0,s0) ,注意 C ⊆ G ′ C\subseteq G' CG ,由于P1和P1加上 s 0 ′ s_0' s0

是最短路径,或P2和P2加上 s 0 s_0 s0 是最短路径,故我们有 r − 1 < = d ′ r-1<=d' r1<=d

另外 d [ s 0 , s 0 ′ , i ] ≠ ⊘ 0 < = i < = d ′ d [ s 0 , s 0 ′ , i ] ∩ d [ s 0 , s 0 ′ , j ] = ⊘ 0 < = i < j < = d ′ 且 V ′ = ⋃ i = 0 d ′ d [ s 0 , s 0 ′ , i ] d[s_{0}, s_{0}',i]\ne \oslash 0<=i<=d'd[s_{0},s_{0}',i]\cap d[s_{0},s_{0}',j] = \oslash0<=i<j<=d' 且 V' = \bigcup_{i = 0}^{d'}{d[s_{0},s_{0}', i]} d[s0,s0,i]=0<=i<=dd[s0,s0,i]d[s0,s0,j]=0<=i<j<=dV=i=0dd[s0,s0,i]

于是类似的有命题 P ′ ( i ) P'(i) P(i)( 0 < = i < r − 1 0<=i<r-1 0<=i<r1)

对于 d [ s 0 , s 0 ′ , i ] d[s_{0},s_{0}',i] d[s0,s0,i] 中的任意顶点 v v v,与 v v v关联的所有边中只有一条和 d [ s 0 , s 0 ′ , i − 1 ] d[s_{0},s_{0}',i-1] d[s0,s0,i1] 中的顶点相连(如果存在,即 i i i不等于0,如果 i = 0 i=0 i=0,由于 s 0 s_0 s0 s 0 ′ s'_0 s0 相互关联,对于任意 t ∈ d [ s 0 , s 0 ′ , 0 ] t \in d[s_{0},s_{0}',0] td[s0,s0,0] ,与其关联的所有顶点中除一个在 d [ s 0 , s 0 ′ , 0 ] d[s_{0},s_{0}',0] d[s0,s0,0] ,其余均在 d [ s 0 , s 0 ′ , 1 ] d[s_{0},s_{0}',1] d[s0,s0,1] 中,如下所述),其余(如果有)全部和 d [ s 0 , s 0 ′ , i + 1 ] d[s_{0},s_{0}',i+1] d[s0,s0,i+1] 中的顶点相连,并且对任意 s 1 s 2 ∈ d [ s 0 , s 0 ′ , i ] s_1s_2\in d[s_{0},s_{0}',i] s1s2d[s0,s0,i] ( s 1 ≠ s 2 s_1 \ne s_2 s1=s2) 有 ( N G ′ ( s 1 ) − ( s 1 ′ ) ) ∩ ( N G ′ ( s 2 ) − ( s 2 ′ ) ) = ⊘ (N_{G'}(s_1)-(s'_1))\cap (N_{G'}(s_2)-(s'_2))= \oslash (NG(s1)(s1))(NG(s2)(s2))= 其中 s 1 ′ s'_1 s1 是与 s 1 s_1 s1 d [ s 0 , s 0 ′ , i − 1 ] d[s_{0},s_{0}',i-1] d[s0,s0,i1] 中相邻的顶点, s 2 ′ s'_2 s2 同理,(当 i = 0 i=0 i=0时, s 1 ′ s'_1 s1 即为 s 2 s_2 s2 , s 2 ′ s'_2 s2 即为 s 1 s_1 s1 )

现在来导出练习所要求的第二个结论

事实上, s 0 , s 0 ′ s_0 , s_0' s0,s0 是我们首先要考虑的顶点其个数为2, ∣ d [ s 0 , s 0 ′ , 0 ] ∣ = 2 |d[s_0,s_0',0]|=2 d[s0,s0,0]=2

由命题 p ( 0 ) p(0) p(0),对任意 v ∈ d [ s 0 , s 0 ′ , 0 ] v \in d[s_0,s_0',0] vd[s0,s0,0] , v v v的所有邻接顶点除一个位于 d [ s 0 , s 0 ′ , 0 ] d[s_0,s_0',0] d[s0,s0,0] 其余均位于 d [ s 0 , s 0 ′ , 1 ] d[s_0,s_0',1] d[s0,s0,1] 中,并且这些位于 d [ s 0 , s 0 ′ , 1 ] d[s_0,s_0',1] d[s0,s0,1] 中的顶点对不同的 v v v不相交,故 ∣ d [ s 0 , s 0 ′ , 1 ] ∣ > = 2 ( δ ( G ′ ) − 1 ) > = 2 ( δ ( G ) − 1 ) |d[s_0,s_0',1]|>=2(\delta(G')-1)>=2(\delta(G)-1) d[s0,s0,1]>=2(δ(G)1)>=2(δ(G)1)

2 < = i < = r − 1 2<=i<=r-1 2<=i<=r1,由命题 P ( i − 1 ) P(i-1) P(i1), d [ s 0 , s 0 ′ , i − 1 ] d[s_0,s_0',i-1] d[s0,s0,i1] 中的每一个顶点的邻点集除一个顶点外,其余均位于 d [ s 0 , s 0 ′ , i ] d[s_0,s_0',i] d[s0,s0,i] 中,且不同顶点的邻点集除位于 d [ s 0 , s 0 ′ , i − 2 ] d[s_0,s_0',i-2] d[s0,s0,i2] 中的一个顶点外其余顶点构成的集合互不相交,故

∣ d [ s 0 , s 0 ′ , i ] ∣ > = ∑ v ∈ d [ s 0 , s 0 ′ , i − 1 ] ∣ N G ′ ( v ) − ( v ′ ) ∣ > = ∑ v ∈ d [ s 0 , s 0 ′ , i − 1 ] δ ( G ′ ) − 1 > = ∣ d [ s 0 , s 0 ′ , i − 1 ] ∣ ( δ ( G ) − 1 ) |d[s_0,s_0', i]|>= \sum_{v\in d[s_0,s_0',i-1]}{|N_{G'}(v)-(v')|}>=\sum_{v\in d[s_0,s_0',i-1]}{\delta(G')-1} >= |d[s_0,s_0',i-1]|(\delta(G)-1) d[s0,s0,i]>=vd[s0,s0,i1]NG(v)(v)>=vd[s0,s0,i1]δ(G)1>=d[s0,s0,i1](δ(G)1)

其中 v ′ v' v v v v d [ s 0 , s 0 ′ , i − 2 ] d[s_0,s_0',i-2] d[s0,s0,i2]中的唯一邻接顶点

从而 ∣ d [ s 0 , s 0 ′ , i ] ∣ > = ∣ d [ s 0 , s 0 ′ , i − 1 ] ∣ ( δ ( G ) − 1 ) > = ∣ d [ s 0 , s 0 ′ , i − 2 ] ∣ ( δ ( G ) − 1 ) 2 > = − − − > = |d[s_0,s_0',i]|>= |d[s_0,s_0',i-1]|(\delta(G)-1)>= |d[s_0,s_0',i-2]|(\delta(G)-1)^{2} >=--->= d[s0,s0,i]>=d[s0,s0,i1](δ(G)1)>=d[s0,s0,i2](δ(G)1)2>=>=

∣ d [ s 0 , s 0 ′ , 1 ] ∣ ( δ ( G ) − 1 ) i − 1 > = 2 ( δ ( G ) − 1 ) ( δ ( G ) − 1 ) i − 1 |d[s_0,s_0',1]|(\delta(G)-1)^{i-1} >= 2(\delta(G)-1)(\delta(G)-1)^{i-1} d[s0,s0,1](δ(G)1)i1>=2(δ(G)1)(δ(G)1)i1

于是 V ( G ) > = V ( G ′ ) = ∣ d [ s 0 , s 0 ′ , 0 ] ∣ + + ∣ d [ s 0 , s 0 ′ , 1 ] ∣ + ∑ i = 2 r − 1 ∣ d [ s 0 , s 0 ′ , i ] ∣ + ∑ i = r d ′ ∣ d [ s 0 , s 0 ′ , i ] ∣ V(G)>=V(G')= |d[s_0,s_0',0]|+ +|d[s_0,s_0',1]|+\sum_{i=2}^{r-1}|d[s_0,s_0',i]|+ \sum_{i = r}^{d'}{|d[s_0,s_0',i]|} V(G)>=V(G)=d[s0,s0,0]++d[s0,s0,1]+i=2r1d[s0,s0,i]+i=rdd[s0,s0,i]

> = ∣ d [ s 0 , s 0 ′ , 0 ] ∣ + ∣ d [ s 0 , s 0 ′ , 1 ] ∣ + ∑ i = 2 r − 1 2 ( δ ( G ) − 1 ) ( δ ( G ) − 1 ) i − 1 + ∑ i = r d ′ ∣ d [ s 0 , s 0 ′ , i ] ∣ >= |d[s_0,s_0',0]|+|d[s_0,s_0',1]|+ \sum_{i=2}^{r-1}2(\delta(G)-1)(\delta(G)-1)^{i-1} + \sum_{i = r}^{d'}{|d[s_0,s_0',i]|} >=d[s0,s0,0]+d[s0,s0,1]+i=2r12(δ(G)1)(δ(G)1)i1+i=rdd[s0,s0,i]

> = ∣ d [ s 0 , s 0 ′ , 0 ] ∣ + ∑ i = 1 r − 1 2 ( δ ( G ) − 1 ) ( δ ( G ) − 1 ) i − 1 = 2 + ∑ i = 1 r − 1 2 ( δ ( G ) − 1 ) ( δ ( G ) − 1 ) i − 1 = >= |d[s_0,s_0',0]|+\sum_{i=1}^{r-1}2(\delta(G)-1)(\delta(G)-1)^{i-1} = 2+ \sum_{i=1}^{r-1}2(\delta(G)-1)(\delta(G)-1)^{i-1} = >=d[s0,s0,0]+i=1r12(δ(G)1)(δ(G)1)i1=2+i=1r12(δ(G)1)(δ(G)1)i1=

2 + ∑ i = 1 r − 1 2 ( δ ( G ) − 1 ) i = 2 ∑ i = 0 r − 1 ( δ ( G ) − 1 ) i 2+ \sum_{i=1}^{r-1}2(\delta(G)-1)^{i} = 2\sum_{i=0}^{r-1}(\delta(G)-1)^{i} 2+i=1r12(δ(G)1)i=2i=0r1(δ(G)1)i

这样就成功导出了练习要求的第二个结论

最后还要就上文第二结论的证明的开头部分说明一下为什么若 d ′ = m a x v ∈ V ′ ( m i n ( d G ′ ( s 0 , v ) , d G ′ ( s 0 ′ , v ) ) ) d'=max_{v\in V'}(min(d_{G'}(s_0,v), d_{G'}(s'_0, v))) d=maxvV(min(dG(s0,v),dG(s0,v))) 则对任意i满足 0 < = i < = d ′ 0<=i<=d' 0<=i<=d

都存在 v ′ ∈ V ′ v'\in V' vV 使得 i = m i n ( d G ′ ( s 0 , v ′ ) , d G ′ ( s 0 ′ , v ′ ) ) i = min(d_{G'}(s_0,v'), d_{G'}(s'_0, v')) i=min(dG(s0,v),dG(s0,v)) ,当 i = d ′ i=d' i=d i = 0 i=0 i=0 时结论是显然的

现考虑 0 < i < d ′ 0<i<d' 0<i<d ,设 m i n ( d G ′ ( s 0 , s ) , d G ′ ( s 0 ′ , s ) ) = d ′ min(d_{G'}(s_0,s), d_{G'}(s'_0, s)) = d' min(dG(s0,s),dG(s0,s))=d

s s s s 0 s_0 s0 的最短路径为P, s s s s 0 ′ s'_0 s0 的最短路径为P’,不妨设 d G ′ ( s 0 , s ) < = d G ′ ( s 0 ′ , s ) d_{G'}(s_0,s)<=d_{G'}(s'_0, s) dG(s0,s)<=dG(s0,s) ,则 d G ′ ( s 0 , s ) = d ′ d_{G'}(s_0,s) = d' dG(s0,s)=d

这样在P的内部顶点中取一点 s’ 使得 d G ′ ( s 0 , s ′ ) = i d_{G'}(s_0,s') = i dG(s0,s)=i,现在可以断言 d G ′ ( s 0 ′ , s ′ ) d_{G'}(s_0',s') dG(s0,s) 不可能小于 i i i,否则有

d G ′ ( s 0 ′ , s ) > = d G ′ ( s 0 , s ) = d G ′ ( s ′ , s ) + d G ′ ( s 0 , s ′ ) = d G ′ ( s ′ , s ) + i > d G ′ ( s ′ , s ) + d G ′ ( s 0 ′ , s ′ ) = L d_{G'}(s'_0, s)>=d_{G'}(s_0,s)=d_{G'}(s',s)+d_{G'}(s_0,s')=d_{G'}(s',s)+i>d_{G'}(s',s)+d_{G'}(s_0',s')=L dG(s0,s)>=dG(s0,s)=dG(s,s)+dG(s0,s)=dG(s,s)+i>dG(s,s)+dG(s0,s)=L

注意到P上 s s s s ′ s' s 的最短路径和 s ′ s' s s 0 ′ s_0' s0 的最短路径连接起来形成的道路对应的路径长度小于等于 L L L ,这和 P ′ P' P s s s s 0 ′ s'_0 s0 的最短路径矛盾

因此 d G ′ ( s 0 ′ , s ′ ) > = i = d G ′ ( s 0 , s ′ ) , 故 m i n ( d G ′ ( s 0 , s ′ ) , d G ′ ( s 0 ′ , s ′ ) ) = d G ′ ( s 0 , s ′ ) = i d_{G'}(s_0',s')>=i=d_{G'}(s_0,s') ,故 min(d_{G'}(s_0,s'),d_{G'}(s_0',s'))=d_{G'}(s_0,s')=i dG(s0,s)>=i=dG(s0,s),min(dG(s0,s),dG(s0,s))=dG(s0,s)=i

v ′ = s ′ v' = s' v=s 即证

评论
添加红包

请填写红包祝福语或标题

红包个数最小为10个

红包金额最低5元

当前余额3.43前往充值 >
需支付:10.00
成就一亿技术人!
领取后你会自动成为博主和红包主的粉丝 规则
hope_wisdom
发出的红包
实付
使用余额支付
点击重新获取
扫码支付
钱包余额 0

抵扣说明:

1.余额是钱包充值的虚拟货币,按照1:1的比例进行支付金额的抵扣。
2.余额无法直接购买下载,可以购买VIP、付费专栏及课程。

余额充值